Diễn Đàn MathScopeDiễn Đàn MathScope
  Diễn Đàn MathScope
Ghi Danh Hỏi/Ðáp Thành Viên Social Groups Lịch Ðánh Dấu Ðã Ðọc

Go Back   Diễn Đàn MathScope > Sơ Cấp > Việt Nam và IMO > 2015

News & Announcements

Ngoài một số quy định đã được nêu trong phần Quy định của Ghi Danh , mọi người tranh thủ bỏ ra 5 phút để đọc thêm một số Quy định sau để khỏi bị treo nick ở MathScope nhé !

* Nội quy MathScope.Org

* Một số quy định chung !

* Quy định về việc viết bài trong diễn đàn MathScope

* Nếu bạn muốn gia nhập đội ngũ BQT thì vui lòng tham gia tại đây

* Những câu hỏi thường gặp

* Về việc viết bài trong Box Đại học và Sau đại học


Trả lời Gởi Ðề Tài Mới
 
Ðiều Chỉnh Xếp Bài
Old 25-03-2015, 12:25 PM   #1
huynhcongbang
Administrator

 
huynhcongbang's Avatar
 
Tham gia ngày: Feb 2009
Đến từ: Ho Chi Minh City
Bài gởi: 2,413
Thanks: 2,165
Thanked 4,188 Times in 1,381 Posts
Gửi tin nhắn qua Yahoo chát tới huynhcongbang
Việt Nam TST 2015 - Đề thi, lời giải và danh sách đội tuyển

Sáng nay, kỳ thi chọn đội tuyển Toán quốc gia dự thi Olympic Toán quốc tế diễn ra tại trường ĐHSP Hà Nội. Hiện giờ, các thí sinh đang bước vào các phút cuối giờ của tổng cộng 270 phút làm bài. Đây có thể nói là kỳ thi Olympic trong nước lớn nhất và căng thẳng nhất.

Năm nay, trên diễn đàn Mathscope không diễn ra các topic thường niên "Hướng tới kỳ thi TST..." và trong TPHCM cũng không tổ chức kịp trường Xuân để hỗ trợ ôn tập, rèn luyện cho một số học sinh trong khu vực miền Trung, miền Nam được vào vòng 2. Đây thực sự là một điều rất đáng tiếc.

Kỳ thi lần này chọn ra 6 thí sinh cho đội tuyển quốc gia để tham dự IMO 2015 tại Chiang Mai, Thái Lan vào đầu tháng 7.

Lát nữa khi kết thúc ngày thi thứ nhất, mong rằng các thầy, các thí sinh có tham gia trực tiếp có thể chia sẻ nội dung đề thi để mọi người có thể tham khảo, phân tích và thử sức mình.

Dưới đây là vài hình ảnh của ngày khai mạc hôm qua mình tìm được trên facebook của một số thí sinh:




[RIGHT][I][B]Nguồn: MathScope.ORG[/B][/I][/RIGHT]
 
Hình Kèm Theo
Kiểu File : jpg 10881708_375567915962661_7073095025052343379_n.jpg (73.0 KB, 1632 lần tải)
Kiểu File : jpg 10923279_435186103317354_2695238345505790779_n.jpg (83.7 KB, 1559 lần tải)
__________________
Sự im lặng của bầy mèo

thay đổi nội dung bởi: huynhcongbang, 25-03-2015 lúc 12:28 PM
huynhcongbang is offline   Trả Lời Với Trích Dẫn
The Following 7 Users Say Thank You to huynhcongbang For This Useful Post:
buigiahuy0 (25-03-2015), khi gia (27-03-2015), n.v.thanh (27-03-2015), pco (25-03-2015), quocbaoct10 (25-03-2015), Short_list (25-03-2015), whatever2507 (25-03-2015)
Old 25-03-2015, 12:34 PM   #2
Short_list
+Thành Viên+
 
Tham gia ngày: May 2012
Đến từ: Tp.HCM
Bài gởi: 85
Thanks: 12
Thanked 79 Times in 32 Posts
Năm nay VMO, TST có vẻ không sôi động bằng mọi năm nhỉ.
[RIGHT][I][B]Nguồn: MathScope.ORG[/B][/I][/RIGHT]
 
__________________
The Simplest Solution Is The Best Solution
Short_list is offline   Trả Lời Với Trích Dẫn
Old 25-03-2015, 01:53 PM   #3
huynhcongbang
Administrator

 
huynhcongbang's Avatar
 
Tham gia ngày: Feb 2009
Đến từ: Ho Chi Minh City
Bài gởi: 2,413
Thanks: 2,165
Thanked 4,188 Times in 1,381 Posts
Gửi tin nhắn qua Yahoo chát tới huynhcongbang
Dưới đây là câu 1 và câu 3 của đề thi mình được thầy trò Trần Quốc Luật (thaygiaocht) chia sẻ:

Bài 1.
Gọi $\alpha $ là nghiệm dương của phương trình ${{x}^{2}}+x=5$. Với số nguyên dương $n$ nào đó, gọi ${{c}_{0}},{{c}_{1}},{{c}_{2}}, \ldots ,{{c}_{n}}$ là các số nguyên không âm thỏa mãn đẳng thức $${{c}_{0}}+{{c}_{1}}\alpha +{{c}_{2}}{{\alpha }^{2}}+...+{{c}_{n}}{{\alpha }^{n}}=2015.$$
a) Chứng minh rằng ${{c}_{0}}+{{c}_{1}}+{{c}_{2}}+...+{{c}_{n}}\equiv 2\text{ }(\bmod 3).$
b) Tìm giá trị nhỏ nhất của tổng ${{c}_{0}}+{{c}_{1}}+{{c}_{2}}+...+{{c}_{n}}$.
Bài 2.
cho đường tròn (O), dây cung $BC$ cố định và điểm $A$ chạy trên $(O)$. Gọi $I,H$ lần lượt là trung điểm cạnh $BC$ và trực tâm tam giác $ABC$, tia $IH$ cắt $(O)$ tại $K$, $AH$ cắt $BC$ tại $D$, $KD$ cắt $(O)$ tại $M$. Từ M vẽ đường vuông góc với $BC$ cắt $AI$ tại $N$.
a) Cmr: $N$ thuộc đường tròn cố định.
b) Đường tròn tiếp xúc với $AK$ tại $A$ và đi qua $N$ cắt $AB,AC$ tại $P,Q$. J là trung điểm $P,Q$. Cmr: $AJ$ qua điểm cố định.
Bài 3.
Một số nguyên dương $k$ có tính chất “$t-m$” nếu với mọi số nguyên dương $a$, tồn tại số nguyên dương $n$ sao cho
$${{1}^{k}}+{{2}^{k}}+{{3}^{k}}+...+{{n}^{k}} \equiv a (\bmod m).$$
a) Tìm tất cả các số nguyên dương $k$ có tính chất $t-20.$
b) Tìm số nguyên dương $k$ nhỏ nhất có tính chất $t-{{20}^{15}}$.
[RIGHT][I][B]Nguồn: MathScope.ORG[/B][/I][/RIGHT]
 
__________________
Sự im lặng của bầy mèo

thay đổi nội dung bởi: quocbaoct10, 25-03-2015 lúc 10:45 PM
huynhcongbang is offline   Trả Lời Với Trích Dẫn
The Following 11 Users Say Thank You to huynhcongbang For This Useful Post:
dangvip123tb (01-04-2015), n.t.tuan (26-03-2015), pco (25-03-2015), quocbaoct10 (26-03-2015), Raul Chavez (25-03-2015), sieusieu90 (26-03-2015), thaygiaocht (25-03-2015), thiendieu96 (25-03-2015), Trieu123 (05-04-2015), vinhhop.qt (26-03-2015), whatever2507 (25-03-2015)
Old 25-03-2015, 02:02 PM   #4
thaygiaocht
+Thành Viên+
 
thaygiaocht's Avatar
 
Tham gia ngày: Aug 2012
Đến từ: Chuyên Hà Tĩnh
Bài gởi: 165
Thanks: 793
Thanked 216 Times in 93 Posts
Đề ngày 1 đẹp nhưng không có bài vừa (dễ thì dễ quá mà khó thì cũng khó quá), ngày mai có lẽ Giải tích - Tổ - Hình.
[RIGHT][I][B]Nguồn: MathScope.ORG[/B][/I][/RIGHT]
 
__________________
https://www.facebook.com/thaygiaocht

thay đổi nội dung bởi: thaygiaocht, 25-03-2015 lúc 05:23 PM
thaygiaocht is offline   Trả Lời Với Trích Dẫn
The Following 3 Users Say Thank You to thaygiaocht For This Useful Post:
dangvip123tb (01-04-2015), huynhcongbang (25-03-2015), thiendieu96 (25-03-2015)
Old 25-03-2015, 02:12 PM   #5
huynhcongbang
Administrator

 
huynhcongbang's Avatar
 
Tham gia ngày: Feb 2009
Đến từ: Ho Chi Minh City
Bài gởi: 2,413
Thanks: 2,165
Thanked 4,188 Times in 1,381 Posts
Gửi tin nhắn qua Yahoo chát tới huynhcongbang
Ủng hộ trước câu 1a khá nhẹ nhàng.

Xét đa thức $$P(x)={{c}_{0}}+{{c}_{1}}x+{{c}_{2}}{{x}^{2}}+... +{{c}_{n}}{{x}^{n}}$$ và xét phép chia đa thức
$$P(x)=({{x}^{2}}+x-5)Q(x)+R(x)$$ với $R(x)$ có hệ số nguyên có bậc bé hơn 2.

Ta thấy $$P(\alpha )=R(\alpha )=2015$$ và $$P(1)=-3Q(1)+R(1)\equiv R(1)\text{ }(\bmod 3).$$ Do đó, ta cần chứng minh $R(1)\equiv 2\text{ }(\bmod 3).$

Nếu $\deg R(x)=1$ thì $R(x)$ có dạng $ax+b$ và $a\alpha +b=2015$. Tuy nhiên, dễ thấy $\alpha $ vô tỉ nên $a\alpha +b$ cũng vô tỉ, mâu thuẫn.

Do đó $\deg R(x)=0$ và suy ra $R(x)=2015\equiv 2(\bmod 3).$
Ta có đpcm.
[RIGHT][I][B]Nguồn: MathScope.ORG[/B][/I][/RIGHT]
 
__________________
Sự im lặng của bầy mèo
huynhcongbang is offline   Trả Lời Với Trích Dẫn
The Following 10 Users Say Thank You to huynhcongbang For This Useful Post:
blackholes. (25-03-2015), dangvip123tb (01-04-2015), DenisO (25-03-2015), n.v.thanh (27-03-2015), Nvthe_cht. (25-03-2015), quocbaoct10 (25-03-2015), thank_you (06-04-2015), thaygiaocht (25-03-2015), thiendieu96 (25-03-2015), vantienducdh (25-03-2015)
Old 25-03-2015, 04:41 PM   #6
huynhcongbang
Administrator

 
huynhcongbang's Avatar
 
Tham gia ngày: Feb 2009
Đến từ: Ho Chi Minh City
Bài gởi: 2,413
Thanks: 2,165
Thanked 4,188 Times in 1,381 Posts
Gửi tin nhắn qua Yahoo chát tới huynhcongbang
Tiếp câu 1b, cách hơi xấu.

Theo câu a, suy ra $P(1)=-3Q(1)+2015$.
Do đó, để tìm GTNN của $P(1)$, ta cần tìm GTLN của $Q(1)$.

Đặt $Q(x)={{b}_{0}}+{{b}_{1}}x+{{b}_{2}}{{x}^{2}}+...+ {{b}_{n-2}}{{x}^{n-2}}$ với ${{b}_{0}},{{b}_{1}},...,{{b}_{n-2}}\in \mathbb{Z}$ thì
$$P(x)=({{x}^{2}}+x-5)Q(x)+2015=({{x}^{2}}+x-5)({{b}_{0}}+{{b}_{1}}x+{{b}_{2}}{{x}^{2}}+b_3x^3. ..+{{b}_{n-2}}{{x}^{n-2}})+2015$$ Ta thực hiện đồng nhất hệ số:
$\begin{align}
& {{a}_{0}}=-5{{b}_{0}}+2015\ge 0 \\
& {{a}_{1}}={{b}_{0}}-5{{b}_{1}}\ge 0 \\
& {{a}_{2}}={{b}_{0}}+{{b}_{1}}-5{{b}_{2}}\ge 0 \\
& {{a}_{3}}={{b}_{1}}+{{b}_{2}}-5{{b}_{3}}\ge 0 \\
& ... \\
& {{a}_{n-2}}={{b}_{n-4}}+{{b}_{n-3}}-5{{b}_{n-2}}\ge 0 \\
& {{a}_{n-1}}={{b}_{n-3}}+{{b}_{n-2}}\ge 0 \\
& {{a}_{n}}={{b}_{n-2}}\ge 0 \\
\end{align}$

Ta cần tìm GTLN của ${{b}_{0}}+{{b}_{1}}+{{b}_{2}}+...+{{b}_{n-2}}$.

Rõ ràng các điều kiện trên cho ta các ràng buộc để tìm được GTLN của từng hệ số của $Q(x).$
Chẳng hạn $b_0 \le \dfrac{2015}{5}=403$ và $b_1 \le \dfrac{403}{5}$ nên chọn $b_1 = 80$, cứ như thế.
Cụ thể là xét dãy số $$\left\{ \begin{align}

& {{b}_{0}}=403,{{b}_{1}}=80, \\
& {{b}_{n}}=\left[ \frac{{{b}_{n-1}}+{{b}_{n-2}}}{5} \right],n\ge 2 \\ \end{align} \right.$$ Rõ ràng dãy này đến 1 lúc nào đó thì bằng 0 nên ta tính được
${{b}_{2}}=96,{{b}_{3}}=35,{{b}_{4}}=26,{{b}_{5}}= 12,{{b}_{6}}=7,{{b}_{7}}=3,{{b}_{8}}=2,{{b}_{9}}=1 ,{{b}_{n}}=0,n\ge 10$.

Suy ra $$Q(1)\le 403+80+96+35+26+12+7+3+2+1=665.$$
Từ đó ta có $P(1)\ge (-3)\cdot 665+2015=20$.
Vậy GTNN cần tìm là 20.
[RIGHT][I][B]Nguồn: MathScope.ORG[/B][/I][/RIGHT]
 
__________________
Sự im lặng của bầy mèo

thay đổi nội dung bởi: huynhcongbang, 25-03-2015 lúc 04:48 PM
huynhcongbang is offline   Trả Lời Với Trích Dẫn
The Following 10 Users Say Thank You to huynhcongbang For This Useful Post:
buigiahuy0 (25-03-2015), dangvip123tb (01-04-2015), duykhanhht (28-03-2015), hieut1k24 (28-03-2015), nhatduyt1k24 (25-03-2015), Nvthe_cht. (25-03-2015), Raul Chavez (25-03-2015), thaygiaocht (25-03-2015), thiendieu96 (25-03-2015), vantienducdh (25-03-2015)
Old 25-03-2015, 08:24 PM   #7
chemthan
Administrator

 
chemthan's Avatar
 
Tham gia ngày: Mar 2009
Bài gởi: 349
Thanks: 0
Thanked 308 Times in 161 Posts
Trích:
Nguyên văn bởi huynhcongbang View Post
Bài 3.
Một số nguyên dương $k$ có tính chất “$t-m$” nếu với mọi số nguyên dương $a$, tồn tại số nguyên dương $n$ sao cho
$${{1}^{k}}+{{2}^{k}}+{{3}^{k}}+...+{{n}^{k}} \equiv a (\bmod m).$$
a) Tìm tất cả các số nguyên dương $k$ có tính chất $t-20.$
b) Tìm số nguyên dương $k$ nhỏ nhất có tính chất $t-{{20}^{15}}$.
Đặt $S_k(n) = 1^k+2^k+3^k+...+n^k$.
Hiển nhiên rằng nếu $k$ có tính chất "$t-m$" thì nó cũng có tính chất "$t-n$" với $n$ là ước của $m$.
Bổ đề 1: Cho $p$ là một số nguyên tố lẻ khi đó $S_k(p)$ không chia hết cho $p$ khi và chỉ khi $(p - 1) | k$.
Chứng minh:
Gọi $r$ là căn nguyên thủy của $p$. Khi đó:
$S_k(p)\equiv r^0 + r^k + ... + r^{(p - 2)k} = \frac{r^{k(p - 1)}-1}{r^k-1}[mod p]$, và từ đó có đpcm.
Cách khác sơ cấp hơn:
Nếu $k < p - 1$. Hiển nhiên đa thức $x^k - 1$ có tối đa $k$ nghiệm theo $mod p$. Do đó tồn tại một số nguyên dương $a$ mà $a^k - 1$ không chia hết cho $p$ và $(a, p) = 1$.
Khi đó $(a, 2a, 3a,..., pa)$ là một hệ thặng dư đầy đủ $mod p$, suy ra:
$a^k + (2a)^k + ... + (pa)^k\equiv S_k(p) [mod p]$.
$\Rightarrow (a^k - 1)S_k(p)\equiv 0[mod p]$.
$\Rightarrow S_k(p) \vdots p$.
Bổ đề 2: Cho $p$ là một số nguyên tố lẻ. Khi đó $k$ có tính chất "$t-p$" khi và chỉ khi $(p - 1) | k$.
Chứng minh:
* Điều kiện cần:
Giả sử $k$ không chia hết cho $p-1$, theo bổ đề 1 thì $S_k(p)$ chia hết cho $p$ (1).
Nếu $k$ lẻ, rõ ràng $S_k(\frac{p - 3}{2})\equiv S_k(\frac{p + 3}{2}) [mod p]$ (2).
Từ (1) và (2) suy ra tập các số dư của $S_k(n)$ khi chia cho $p$ không vượt quá $p - 1$, mâu thuẫn.
Nếu $k$ chẵn, ta có $S_k(\frac{p - 1}{2})\equiv S_k(p)\equiv 0[mod p]$ và từ đây cũng dẫn tới mâu thuẫn.
* Điều kiện đủ là hiển nhiên.
Bổ đề 3: Cho $m, n$ là 2 số nguyên dương nguyên tố cùng nhau. Khi đó nếu $k$ có tính chất "$t-mn$" khi và chỉ khi nó có tính chất "$t-m$" và "$t-n$".
Chứng minh:
* Điều kiện cần là hiển nhiên.
* Điều kiện đủ:
Với một số nguyên dương $a$ bất kì tồn tại $x, y$ thỏa mãn:
$S_k(x)\equiv a[mod m]$.
$S_k(y)\equiv a[mod n]$.
Theo định lý thặng dư Trung Hoa tồn tại $b$ thỏa mãn:
$b\equiv x[mod m^2]$.
$b\equiv y[mod n^2]$.
Khi đó:
$S_k(b)\equiv S_k(x)\equiv a[mod m]$.
$S_k(b)\equiv S_k(y)\equiv a[mod n]$.
Và ta có đpcm.
Trở lại bài toán:
a) $k$ có tính chất "$t-20$" Khi và chỉ khi $k$ sẽ có tính chất "$t-5$" và "$t-4$".
Theo bổ đề 2, $k$ có tính chất "$t-5$" khi và chỉ khi $4 | k$.
Khi đó, ta có:
$S_k(4l)\equiv lS_k(4)\equiv 2l[mod 4]$.
$S_k(4l + 1)\equiv lS_k(4)+ 1\equiv 2l + 1[mod 4]$.
Suy ra $k$ có tính chất "$t-4$".
Do đó $4 | k$ là điều kiện cần và đủ.
b) Theo a) thì $k$ có tính chất "$t-20^{15}$" thì $4 | k$.
Và do đó $k \ge 4$. Ta sẽ chứng minh $k = 4$ là số nhỏ nhất có tính chất "$t-20^{15}$".
Theo bổ đề 3, $k$ có tính chất "$t-20^{15}$" khi và chỉ khi $k$ có tính chất "$t-5^{15}$" và "$t-2^{30}$".
Bằng quy nạp theo $l$ ta chứng minh giả thiết:
Cho $p$ là một số nguyên tố, $p\in \{2, 5\}$. Khi đó $4$ có tính chất "$t-p^l$".
Dễ thấy giả thiết đúng với $l = 1$. Giả sử nó đúng với $l$.
Dựa vào công thức: $S_4(n) = \frac{n(6n^4 + 15n^3 + 10n^2 - 1)}{30}$ suy ra $S_4(p^{l + 1})$ chia hết cho $p^l$ nhưng không chia hết cho $p^{l + 1}$. Ta có:
$S_4(xp^{l + 1} + y) \equiv xS_4(p^{l + 1}) + S_4(y)[mod p^{l + 1}]$.
Vì $S_4(p^{l + 1})$ chia hết cho $p^l$ nhưng không chia hết cho $p^{l + 1}$, 4 có tính chất "$t-p^l$" nên dễ dàng suy ra 4 có tính chất "$t-p^{l + 1}$" và giả thiết được chứng minh.
Thay $p = 2, 5$ vào ta suy ra $4$ có tính chất "$t-20^{15}$".
[RIGHT][I][B]Nguồn: MathScope.ORG[/B][/I][/RIGHT]
 

thay đổi nội dung bởi: chemthan, 25-03-2015 lúc 09:52 PM
chemthan is offline   Trả Lời Với Trích Dẫn
The Following 7 Users Say Thank You to chemthan For This Useful Post:
dangvip123tb (01-04-2015), huynhcongbang (25-03-2015), n.v.thanh (27-03-2015), quocbaoct10 (25-03-2015), Raul Chavez (26-03-2015), thaygiaocht (25-03-2015), vantienducdh (25-03-2015)
Old 25-03-2015, 10:09 PM   #8
hoca
+Thành Viên+
 
Tham gia ngày: Oct 2013
Bài gởi: 55
Thanks: 1
Thanked 13 Times in 8 Posts
Trích:
Nguyên văn bởi vantienducdh View Post
Có bản full đề của ngày hôm nay chưa ạ,nghe nói Việt Hà CHT làm hết,còn lại mọi người đều làm đựơc từ 1 đến 2 bài,tình hình TST năm nay có vẻ căng
Bạn đừng chém thế, mình hỏi nó thấy làm ko tốt lă. mà
[RIGHT][I][B]Nguồn: MathScope.ORG[/B][/I][/RIGHT]
 
hoca is offline   Trả Lời Với Trích Dẫn
Old 25-03-2015, 10:12 PM   #9
quocbaoct10
+Thành Viên Danh Dự+
 
quocbaoct10's Avatar
 
Tham gia ngày: Oct 2012
Đến từ: THPT chuyên Lê Quý Đôn-Nha Trang-Khánh Hòa
Bài gởi: 539
Thanks: 292
Thanked 365 Times in 217 Posts
Bài 3 a có thể giải cách khác như sau.
Ta cần chứng minh $S_k \equiv 2 \pmod{5}$ không xảy ra.
Đặt $t_n=\sum (5n+i)^k$
Từ:
$(5n+i)^k \equiv i^k \pmod{5} \\
n^{4k+i} \equiv n^i \pmod{5} $
Ta nhận thấy được rằng không tồn tại số $k$ để cho $\sum (5n+i)^k \equiv 2 \pmod{p}$.(ta xét với các số $i^k$ với $i=1,2,3,4$ và $k=1,2,3$).
Nên từ đó với mọi $k$ không chia hết cho 4 thì $S_k \equiv 2 \pmod{5}$ không xảy ra.
[RIGHT][I][B]Nguồn: MathScope.ORG[/B][/I][/RIGHT]
 
__________________
i'll try my best.
quocbaoct10 is offline   Trả Lời Với Trích Dẫn
The Following User Says Thank You to quocbaoct10 For This Useful Post:
dangvip123tb (01-04-2015)
Old 25-03-2015, 10:20 PM   #10
n.t.tuan
+Thành Viên+
 
n.t.tuan's Avatar
 
Tham gia ngày: Nov 2007
Bài gởi: 1,250
Thanks: 119
Thanked 616 Times in 249 Posts
Luật có đề bài Hình không?
[RIGHT][I][B]Nguồn: MathScope.ORG[/B][/I][/RIGHT]
 
__________________
T.
n.t.tuan is offline   Trả Lời Với Trích Dẫn
The Following User Says Thank You to n.t.tuan For This Useful Post:
dangvip123tb (01-04-2015)
Old 25-03-2015, 10:35 PM   #11
vantienducdh
+Thành Viên+
 
Tham gia ngày: Aug 2014
Đến từ: 12 Toán THPT chuyên LQĐ-Quảng Trị
Bài gởi: 45
Thanks: 35
Thanked 11 Times in 10 Posts
Trích:
Nguyên văn bởi hoca View Post
Bạn đừng chém thế, mình hỏi nó thấy làm ko tốt lă. mà
mình chỉ nghe nói lại thế thôi
[RIGHT][I][B]Nguồn: MathScope.ORG[/B][/I][/RIGHT]
 
__________________
MỘT BÀI TOÁN HAY LÀ BÀI TOÁN KHÔNG ÁP DỤNG NHIỀU KỸ THUẬT MÀ BÀI TOÁN ĐÓ PHẢI ĐẾN TỰ NHIÊN,DỄ HIỂU NHẤT
vantienducdh is offline   Trả Lời Với Trích Dẫn
Old 25-03-2015, 10:41 PM   #12
quocbaoct10
+Thành Viên Danh Dự+
 
quocbaoct10's Avatar
 
Tham gia ngày: Oct 2012
Đến từ: THPT chuyên Lê Quý Đôn-Nha Trang-Khánh Hòa
Bài gởi: 539
Thanks: 292
Thanked 365 Times in 217 Posts
Câu hình: cho đường tròn (O), dây cung $BC$ cố định và điểm $A$ chạy trên $(O)$. Gọi $I,H$ lần lượt là trung điểm cạnh $BC$ và trực tâm tam giác $ABC$, tia $IH$ cắt $(O)$ tại $K$, $AH$ cắt $BC$ tại $D$, $KD$ cắt $(O)$ tại $M$. Từ M vẽ đường vuông góc với $BC$ cắt $AI$ tại $N$.
a) Cmr: $N$ thuộc đường tròn cố định.
b) Đường tròn tiếp xúc với $AK$ tại $A$ và đi qua $N$ cắt $AB,AC$ tại $P,Q$. J là trung điểm $P,Q$. Cmr: $AJ$ qua điểm cố định.
[RIGHT][I][B]Nguồn: MathScope.ORG[/B][/I][/RIGHT]
 
__________________
i'll try my best.
quocbaoct10 is offline   Trả Lời Với Trích Dẫn
The Following 6 Users Say Thank You to quocbaoct10 For This Useful Post:
dangvip123tb (01-04-2015), huynhcongbang (26-03-2015), n.t.tuan (26-03-2015), thaygiaocht (25-03-2015), thiendieu96 (27-03-2015), vantienducdh (25-03-2015)
Old 26-03-2015, 01:19 AM   #13
vinhhop.qt
+Thành Viên+
 
Tham gia ngày: Mar 2010
Bài gởi: 86
Thanks: 44
Thanked 70 Times in 34 Posts
Trích:
Nguyên văn bởi quocbaoct10 View Post
Câu hình: cho đường tròn (O), dây cung $BC$ cố định và điểm $A$ chạy trên $(O)$. Gọi $I,H$ lần lượt là trung điểm cạnh $BC$ và trực tâm tam giác $ABC$, tia $IH$ cắt $(O)$ tại $K$, $AH$ cắt $BC$ tại $D$, $KD$ cắt $(O)$ tại $M$. Từ M vẽ đường vuông góc với $BC$ cắt $AI$ tại $N$.
a) Cmr: $N$ thuộc đường tròn cố định.
b) Đường tròn tiếp xúc với $AK$ tại $A$ và đi qua $N$ cắt $AB,AC$ tại $P,Q$. J là trung điểm $P,Q$. Cmr: $AJ$ qua điểm cố định.
A) gọi A' là giao của AI với (o). Tam giác NMA' vuông ở M, và có IM=IA' nên IN=IM. Suy ra N đối xứng với M qua BC, suy ra đpcm.
[RIGHT][I][B]Nguồn: MathScope.ORG[/B][/I][/RIGHT]
 
vinhhop.qt is offline   Trả Lời Với Trích Dẫn
Old 26-03-2015, 02:11 AM   #14
huynhcongbang
Administrator

 
huynhcongbang's Avatar
 
Tham gia ngày: Feb 2009
Đến từ: Ho Chi Minh City
Bài gởi: 2,413
Thanks: 2,165
Thanked 4,188 Times in 1,381 Posts
Gửi tin nhắn qua Yahoo chát tới huynhcongbang
Dưới đây là lời giải chi tiết cho bài hình. Theo mình thì bài này tuy có mô hình khá đơn giản nhưng lại kết hợp nhiều kết quả có sẵn, nếu không nắm vững sẽ giải rất vất vả. Mình phải mất gần 2 tiếng mới giải xong bài này.


a) Gọi $E$ là điểm đối xứng với $A$ qua $O$ thì dễ thấy tứ giác $BHCE$ là hình bình hành và có $K,H,I,E$ thẳng hàng.

Do đó $\angle AKI=\angle ADI=90{}^\circ $ nên tứ giác $AKDI$ nội tiếp. Suy ra $$\angle AIB=180{}^\circ -\angle AKD=\angle ACM.$$ Ngoài ra, $\angle ABI=\angle AMC$ nên hai tam giác $ABI,AMC$ đồng dạng, suy ra $\angle BAI=\angle MAC$.

Gọi $F$ là giao điểm của trung tuyến $AI$ với $(O)$ thì từ các tam giác đồng dạng, ta có $$\angle FCB=\angle BAI=\angle MAC=\angle CBM.$$ Do đó, $BM=CF$ hay $MF\parallel BC.$

Suy ra $IM=IF=IN$ nên $BC$đi qua trung điểm của $MN$ hay nói cách khác, $M,N$ đối xứng nhau qua $BC$.

Vậy $\angle BNC=\angle BMC=180{}^\circ -\angle BAC$ cố định nên điểm $N$ di chuyển trên cung chứa góc $180{}^\circ -\angle BAC$ dựng trên $BC$, tức là thuộc đường tròn cố định.

b) Gọi $(S)$ là đường tròn ngoại tiếp tam giác $APQ.$ Giả sử $HN$ cắt $(S)$ tại $R$.

Tứ giác $HNEJ$ có hai đường chéo cắt nhau tại trung điểm mỗi đường nên là hình bình hành, suy ra $HN\parallel JE$, mà $JE\bot AJ$ nên $HN\parallel AJ.$ Do đó, tứ giác $HNID$ nội tiếp.

Ta có $$\angle AQN=\angle ARN=\angle NHI=\angle NDI$$ nên $CQND$ nội tiếp.

Suy ra $$\angle NQD=\angle NCD=\angle DCM=\angle BAM=\angle CAJ=\angle QAN$$ hay $DQ$ là tiếp tuyến của đường tròn $(S).$
Tương tự thì $DP$ cũng là tiếp tuyến của $(S)$ nên $AD$ là đường đối trung của tam giác $APQ$, tức là $AJ,AD$ đối xứng nhau qua phân giác góc $\angle BAC.$

Hơn nữa, do $H,O$ lần lượt là trực tâm và tâm đường tròn ngoại tiếp tam giác $ABC$ nên $AH,AO$ cũng đối xứng nhau qua phân giác góc $\angle BAC$.

Từ đây suy ra $AJ,AO$ trùng nhau hay $AJ$ đi qua điểm $O$ cố định.
[RIGHT][I][B]Nguồn: MathScope.ORG[/B][/I][/RIGHT]
 
Hình Kèm Theo
Kiểu File : png abc.png (60.3 KB, 1464 lần tải)
__________________
Sự im lặng của bầy mèo

thay đổi nội dung bởi: huynhcongbang, 26-03-2015 lúc 09:40 AM
huynhcongbang is offline   Trả Lời Với Trích Dẫn
The Following 5 Users Say Thank You to huynhcongbang For This Useful Post:
alibaba_cqt (31-03-2015), dangvip123tb (01-04-2015), quocbaoct10 (26-03-2015), thiendieu96 (27-03-2015), vantienducdh (26-03-2015)
Old 26-03-2015, 06:11 AM   #15
quocbaoct10
+Thành Viên Danh Dự+
 
quocbaoct10's Avatar
 
Tham gia ngày: Oct 2012
Đến từ: THPT chuyên Lê Quý Đôn-Nha Trang-Khánh Hòa
Bài gởi: 539
Thanks: 292
Thanked 365 Times in 217 Posts
Nhận xét một chút về bài 3:
Từ các con số $20$ hay $20^{15}$, ta hoàn toàn có thể tổng quát nó lên thành các số $m=a.b$ với $a=2^k$ và $b$ là một số có căn nguyên thủy, cụ thể $2,4,p^k,2p^k$. Lời giải Hoàn toàn có thể xây dựng dựa vào bổ đề 1 trong lời giải của anh Trung :"Cho $p$ là một số nguyên tố lẻ khi đó $S_k(p)$ không chia hết cho $p$ khi và chỉ khi $(p - 1) | k$". Và có một câu hỏi được đặt ra: liệu với $m$ là tích của 2 số có căn nguyên thủy( 2 số này khác $2$ và $4$) thì có tồn tại $k \neq 1$ sao cho $k$ mang tính chất $t-m$ hay không ? Và nếu chứng minh được điều này thì việc mở rộng ra với tất cả các số tự nhiên là hoàn toàn có thể, dựa vào bổ đề 3 của anh Trung: "Cho $m, n$ là 2 số nguyên dương nguyên tố cùng nhau. Khi đó nếu $k$ có tính chất "$t-mn$" khi và chỉ khi nó có tính chất "$t-m$" và "$t-n$" ".
[RIGHT][I][B]Nguồn: MathScope.ORG[/B][/I][/RIGHT]
 
__________________
i'll try my best.

thay đổi nội dung bởi: quocbaoct10, 26-03-2015 lúc 06:25 AM
quocbaoct10 is offline   Trả Lời Với Trích Dẫn
The Following 5 Users Say Thank You to quocbaoct10 For This Useful Post:
dangvip123tb (01-04-2015), Raul Chavez (26-03-2015), thaygiaocht (26-03-2015), thiendieu96 (27-03-2015), vantienducdh (26-03-2015)
Trả lời Gởi Ðề Tài Mới

Bookmarks

Ðiều Chỉnh
Xếp Bài

Quuyền Hạn Của Bạn
You may not post new threads
You may not post replies
You may not post attachments
You may not edit your posts

BB code is Mở
Smilies đang Mở
[IMG] đang Mở
HTML đang Tắt

Chuyển đến


Múi giờ GMT. Hiện tại là 11:01 AM.


Powered by: vBulletin Copyright ©2000-2024, Jelsoft Enterprises Ltd.
Inactive Reminders By mathscope.org
[page compression: 122.23 k/139.20 k (12.19%)]